Difference between revisions of "2009 AMC 10B Problems/Problem 22"

(Problem)
(Solution 2)
(43 intermediate revisions by 5 users not shown)
Line 57: Line 57:
 
</asy>
 
</asy>
  
Let's label the points as in the picture above. Let <math>[RNQ]</math> be the area of <math>\triangle RNQ</math>. Then the volume of the corresponding piece is <math>c=2[RNQ]</math>. This cake piece has icing on the top and on the vertical side that contains the edge <math>QR</math>. Hence the total area with icing is <math>[RNQ]+2^2 = [RNQ]+4</math>. Thus the answer to our problem is <math>3[RNQ]+4</math>, and all we have to do now is to determine <math>[RNQ]</math>.
+
Let's label the points as in the picture above. Let <math>[RNQ]</math> be the area of <math>\triangle RNQ</math>. Then the volume of the corresponding piece is <math>c=2[RNQ]</math>. This cake piece has icing on the top and on the vertical side that contains the edge <math>QR</math>. Hence the total area with icing is <math>[RNQ]+2^2 = [RNQ]+4</math>. Thus the answer to our problem is <math>3[RNQ]+4</math>, and all we have to do now is determine <math>[RNQ]</math>.
  
 
=== Solution 1 ===
 
=== Solution 1 ===
Line 98: Line 98:
 
</asy>
 
</asy>
  
It is now obvious that <math>O</math> is the midpoint of <math>PQ</math>. (Imagine rotating the square <math>PQRS</math> by <math>90^\circ</math> clockwise around its center. This rotation will map the segment <math>MQ</math> to a segment that is orthogonal to <math>MQ</math>, contains <math>R</math> and contains the midpoint of <math>PQ</math>.)
+
It is now obvious that <math>O</math> is the midpoint of <math>PQ</math>. (Imagine rotating the square <math>PQRS</math> by <math>90^\circ</math> clockwise around its center. This rotation will map the segment <math>MQ</math> to a segment that is orthogonal to <math>MQ</math>, contains <math>R</math> and contains the midpoint of <math>PQ</math>.
  
 
From <math>\triangle PQM</math> we can compute that <math>QM = \sqrt{1^2 + 2^2} = \sqrt 5</math>.
 
From <math>\triangle PQM</math> we can compute that <math>QM = \sqrt{1^2 + 2^2} = \sqrt 5</math>.
Line 108: Line 108:
 
Knowing this, we can compute the area of <math>\triangle NQO</math> as <math>[NQO] = \frac{ON \cdot NQ}2 = \frac 15</math>.
 
Knowing this, we can compute the area of <math>\triangle NQO</math> as <math>[NQO] = \frac{ON \cdot NQ}2 = \frac 15</math>.
  
Finally, we compute <math>[RNQ] = [ROQ] - [NQO] = 1 - \frac 15 = \frac 45</math>, and conclude that the answer is <math>3[RNQ]+4 = 3\cdot \frac 45 + 4 = \boxed{\frac{32}5}</math>.
+
Finally, we compute <math>[RNQ] = [ROQ] - [NQO] = 1 - \frac 15 = \frac 45,</math> and conclude that the answer is <math>3[RNQ]+4 = 3\cdot \frac 45 + 4 = \boxed{\frac{32}5}.</math>
  
*You could also notice that the two triangles in the original figure are similar.
+
*You could also notice that the two triangles <math>\triangle PMQ</math> and <math>\triangle NQR</math> in the original figure are similar.
  
=== Solution 3 ===
+
==Solution 3 (Pythagorean Theorem only)==
 
 
Use trigonometry.
 
 
 
The length of <math>PM</math> and <math>PQ</math> is <math>1</math> and <math>2</math> respectively. So <math>\angle PQM = \arctan \frac 12</math>, and <math>\angle NQR = 90 - \angle PQM = 90 - \arctan \frac 12</math>.
 
 
 
From the right-angled triangle <math>\triangle NQR</math>, the hypotenuse, <math>QR = 2</math> So <math>NR = 2 \sin (90 - \arctan \frac 12)</math>, and <math>NQ = 2 \cos (90 - \arctan \frac 12)</math>
 
 
 
Knowing this, <math>[RNQ] = \frac 12 \cdot NR \cdot NQ</math>. So we proceed as follows:
 
 
 
<math>[RNQ] = \frac 12 \cdot NR \cdot NQ</math>
 
 
 
<math>[RNQ] = \frac 12 \cdot 2 \sin (90 - \arctan \frac 12) \cdot 2 \cos (90 - \arctan \frac 12)</math>
 
 
 
<math>[RNQ] = 2 \sin (90 - \arctan \frac 12) \cos (90 - \arctan \frac 12)</math>
 
 
 
<math>[RNQ] = \sin [2(90 - \arctan \frac 12)] = \sin (2 \arctan \frac 12)</math>
 
 
 
<math>[RNQ] = \frac{2 \cdot \frac 12}{1+ (\frac 12)^2}</math>
 
 
 
<math>[RNQ] = \frac 45</math>
 
 
 
So the answer is <math>3[RNQ]+4 = 3 \cdot \frac 45 + 4 = \boxed{\frac{32}5}</math>.
 
 
 
''Note that we didn't use a calculator, but we used trigonometric identities''
 
 
 
==Solution 4 (Pythagorean Theorem only)==
 
  
 
<asy>
 
<asy>
Line 164: Line 138:
 
</asy>
 
</asy>
  
Since <math>PQ = SR = 2</math> and <math>PM = MS = 1</math>, we know that <math>MQ = MR = \sqrt{2^{1} + 1^{2}} = \sqrt{5}</math>.  If we let <math>NQ = x</math>, then <math>MN = \sqrt{5} - x</math>.  Now, by the Pythagorean Theorem, we have:
+
Since <math>PQ = SR = 2</math> and <math>PM = MS = 1</math>, we know that <math>MQ = MR = \sqrt{2^{2} + 1^{2}} = \sqrt{5}</math>.  If we let <math>NQ = x</math>, then <math>MN = \sqrt{5} - x</math>.  Now, by the Pythagorean Theorem, we have:
  
 
<cmath>x^{2} + NR^{2} = 2^{2} = 4</cmath>
 
<cmath>x^{2} + NR^{2} = 2^{2} = 4</cmath>
Line 187: Line 161:
  
 
Therefore, the area of triangle <math>RNQ</math> is <math>\frac{1}{2} \cdot \frac{2\sqrt{5}}{5} \cdot\frac{4\sqrt{5}}{5} = \frac{1}{2} \cdot\frac{40}{25} = \frac{4}{5}</math>.  Since the solution to the problem is <math>3[RNQ] + 4</math>, the answer is <math>3(\frac{4}{5}) + 4 = \frac{12}{5} + \frac{20}{5} = \boxed{(B) \frac{32}{5}}</math>.
 
Therefore, the area of triangle <math>RNQ</math> is <math>\frac{1}{2} \cdot \frac{2\sqrt{5}}{5} \cdot\frac{4\sqrt{5}}{5} = \frac{1}{2} \cdot\frac{40}{25} = \frac{4}{5}</math>.  Since the solution to the problem is <math>3[RNQ] + 4</math>, the answer is <math>3(\frac{4}{5}) + 4 = \frac{12}{5} + \frac{20}{5} = \boxed{(B) \frac{32}{5}}</math>.
 +
 +
==Solution 4==
 +
 +
<asy>
 +
unitsize(2cm);
 +
defaultpen(linewidth(.8pt)+fontsize(8pt));
 +
 +
draw((-1,-1)--(1,-1)--(1,1)--(-1,1)--cycle);
 +
draw((1,1)--(-1,0));
 +
pair P=foot((1,-1),(1,1),(-1,0));
 +
draw((1,-1)--P);
 +
draw(rightanglemark((-1,0),P,(1,-1),4));
 +
 +
label("$M$",(-1,0),W);
 +
label("$C$",(-0.1,-0.3));
 +
label("$A$",(-0.4,0.7));
 +
label("$B$",(0.7,0.4));
 +
label("$P$",(-1,1),NW);
 +
label("$Q$",(1,1),NE);
 +
label("$R$",(1,-1),SE);
 +
label("$S$",(-1,-1),SW);
 +
label("$N$",P,NW);
 +
</asy>
 +
 +
<math>MQ = \sqrt{2^{2} + 1^{2}} = \sqrt{5}</math>
 +
 +
since <math>\angle PQM + \angle PMQ = 90 = \angle PQM + \angle NQR</math>
 +
 +
therefore <math>\angle PMQ = \angle NQR</math>
 +
 +
and since <math>\angle MPQ = \angle QNR = 90</math>
 +
 +
therefore <math>\triangle MPQ \sim \triangle QNR</math>
 +
 +
therefore <math>\frac {[QNR]}{[MPQ]} = (\frac{QR}{QM})^{2}, [QNR] = [MPQ] \cdot (\frac{QR}{QM})^{2}</math>
 +
 +
<math>[MPQ] = \frac{1}{2} \cdot 2 \cdot 1 = 1</math>
 +
 +
<math>[QNR] = 1 \cdot (\frac{2}{\sqrt{5}})^{2} = 1 \cdot \frac{4}{5} = \frac{4}{5}</math>
 +
 +
Since the solution to the problem is <math>3[QNR] + 4</math>, the answer is <math>3(\frac{4}{5}) + 4 = \frac{12}{5} + \frac{20}{5} = \boxed{(B) \frac{32}{5}}</math>.
  
 
== See Also ==
 
== See Also ==

Revision as of 13:09, 2 July 2021

Problem

A cubical cake with edge length $2$ inches is iced on the sides and the top. It is cut vertically into three pieces as shown in this top view, where $M$ is the midpoint of a top edge. The piece whose top is triangle $B$ contains $c$ cubic inches of cake and $s$ square inches of icing. What is $c+s$?

[asy] unitsize(1cm); defaultpen(linewidth(.8pt)+fontsize(8pt));  draw((-1,-1)--(1,-1)--(1,1)--(-1,1)--cycle); draw((1,1)--(-1,0)); pair P=foot((1,-1),(1,1),(-1,0)); draw((1,-1)--P); draw(rightanglemark((-1,0),P,(1,-1),4));  label("$M$",(-1,0),W); label("$C$",(-0.1,-0.3)); label("$A$",(-0.4,0.7)); label("$B$",(0.7,0.4)); [/asy]

$\text{(A) } \frac{24}{5} \qquad \text{(B) } \frac{32}{5} \qquad \text{(C) } 8+\sqrt5 \qquad \text{(D) } 5+\frac{16\sqrt5}{5} \qquad \text{(E) } 10+5\sqrt5$

Solution

[asy] unitsize(2cm); defaultpen(linewidth(.8pt)+fontsize(8pt));  draw((-1,-1)--(1,-1)--(1,1)--(-1,1)--cycle); draw((1,1)--(-1,0)); pair P=foot((1,-1),(1,1),(-1,0)); draw((1,-1)--P); draw(rightanglemark((-1,0),P,(1,-1),4));  label("$M$",(-1,0),W); label("$C$",(-0.1,-0.3)); label("$A$",(-0.4,0.7)); label("$B$",(0.7,0.4)); label("$P$",(-1,1),NW); label("$Q$",(1,1),NE); label("$R$",(1,-1),SE); label("$S$",(-1,-1),SW); label("$N$",P,NW); [/asy]

Let's label the points as in the picture above. Let $[RNQ]$ be the area of $\triangle RNQ$. Then the volume of the corresponding piece is $c=2[RNQ]$. This cake piece has icing on the top and on the vertical side that contains the edge $QR$. Hence the total area with icing is $[RNQ]+2^2 = [RNQ]+4$. Thus the answer to our problem is $3[RNQ]+4$, and all we have to do now is determine $[RNQ]$.

Solution 1

Introduce a coordinate system where $Q=(0,0)$, $P=(2,0)$ and $R=(0,2)$.

In this coordinate system we have $M=(2,1)$, and the line $QM$ has the equation $2y-x=0$.

As the line $RN$ is orthogonal to $QM$, it must have the equation $y+2x+q=0$ for some suitable constant $q$. As this line contains the point $R=(0,2)$, we have $q=-2$.

Substituting $x=2y$ into $y+2x-2=0$, we get $y=\frac 25$, and then $x=\frac 45$.

We can note that in $\triangle RNQ$ $x$ is the height from $N$ onto $RQ$, hence its area is $[RNQ] = \frac{x \cdot RQ} 2 = \frac{2x}2 = x = \frac 45$, and therefore the answer is $3[RNQ]+4 = 3\cdot \frac 45 + 4 = \boxed{\frac{32}5 \Longrightarrow B}$.

Solution 2

Extend $RN$ to intersect $PQ$ at $O$:

[asy] unitsize(2cm); defaultpen(linewidth(.8pt)+fontsize(8pt));  draw((-1,-1)--(1,-1)--(1,1)--(-1,1)--cycle); draw((1,1)--(-1,0)); pair P=foot((1,-1),(1,1),(-1,0)); draw((1,-1)--P); draw(rightanglemark((-1,0),P,(1,-1),4)); draw(P -- (0,1));  label("$M$",(-1,0),W); label("$C$",(-0.1,-0.3)); label("$A$",(-0.4,0.7)); label("$B$",(0.7,0.4)); label("$P$",(-1,1),NW); label("$Q$",(1,1),NE); label("$R$",(1,-1),SE); label("$S$",(-1,-1),SW); label("$N$",P,1.5*WNW); label("$O$",(0,1),N); [/asy]

It is now obvious that $O$ is the midpoint of $PQ$. (Imagine rotating the square $PQRS$ by $90^\circ$ clockwise around its center. This rotation will map the segment $MQ$ to a segment that is orthogonal to $MQ$, contains $R$ and contains the midpoint of $PQ$.

From $\triangle PQM$ we can compute that $QM = \sqrt{1^2 + 2^2} = \sqrt 5$.

Observe that $\triangle PQM$ and $\triangle NQO$ have the same angles and therefore they are similar. The ratio of their sides is $\frac{QM}{OQ} = \frac{\sqrt 5}1 = \sqrt 5$.

Hence we have $ON = \frac{PM}{\sqrt 5} = \frac 1{\sqrt 5}$, and $NQ = \frac{PQ}{\sqrt 5} = \frac 2{\sqrt 5}$.

Knowing this, we can compute the area of $\triangle NQO$ as $[NQO] = \frac{ON \cdot NQ}2 = \frac 15$.

Finally, we compute $[RNQ] = [ROQ] - [NQO] = 1 - \frac 15 = \frac 45,$ and conclude that the answer is $3[RNQ]+4 = 3\cdot \frac 45 + 4 = \boxed{\frac{32}5}.$

  • You could also notice that the two triangles $\triangle PMQ$ and $\triangle NQR$ in the original figure are similar.

Solution 3 (Pythagorean Theorem only)

[asy] unitsize(2cm); defaultpen(linewidth(.8pt)+fontsize(8pt));  draw((-1,-1)--(1,-1)--(1,1)--(-1,1)--cycle); draw((1,1)--(-1,0)); pair P=foot((1,-1),(1,1),(-1,0)); draw((1,-1)--P); draw(rightanglemark((-1,0),P,(1,-1),4)); draw((-1,0)--(1,-1));  label("$M$",(-1,0),W); label("$C$",(-0.1,-0.3)); label("$A$",(-0.4,0.7)); label("$B$",(0.7,0.4)); label("$P$",(-1,1),NW); label("$Q$",(1,1),NE); label("$R$",(1,-1),SE); label("$S$",(-1,-1),SW); label("$N$",P,NW); label("$x$", (0.65, 0.7)); label("$\sqrt{5} - x$", (-0.3, 0.15)); [/asy]

Since $PQ = SR = 2$ and $PM = MS = 1$, we know that $MQ = MR = \sqrt{2^{2} + 1^{2}} = \sqrt{5}$. If we let $NQ = x$, then $MN = \sqrt{5} - x$. Now, by the Pythagorean Theorem, we have:

\[x^{2} + NR^{2} = 2^{2} = 4\] \[(\sqrt{5} - x)^{2} + NR^{2} = (\sqrt{5})^{2} = 5\]

Expanding and rearranging the second equation gives:

\[5 - 2x\sqrt{5} + x^{2} + NR^{2} = 5\] \[x^{2} + NR^{2} - 2x\sqrt{5} = 0\] \[x^{2} + NR^{2} = 2x\sqrt{5}\]

Since $x^{2} + NR^{2} = 4$, we have that:

\[2x\sqrt{5} = 4\] \[x\sqrt{5} = 2\] \[x = \frac{2}{\sqrt{5}} = \frac{2\sqrt{5}}{5}\]

Knowing $x$, we can solve for the height $NR$:

\[NR^{2} = 2^{2} - x^{2} = 4 - ({\frac{2\sqrt{5}}{5}})^{2} = 4 - \frac{4}{5} = \frac{16}{5}\] \[NR = \sqrt{\frac{16}{5}} = \frac{4}{\sqrt{5}} = \frac{4\sqrt{5}}{5}\]

Therefore, the area of triangle $RNQ$ is $\frac{1}{2} \cdot \frac{2\sqrt{5}}{5} \cdot\frac{4\sqrt{5}}{5} = \frac{1}{2} \cdot\frac{40}{25} = \frac{4}{5}$. Since the solution to the problem is $3[RNQ] + 4$, the answer is $3(\frac{4}{5}) + 4 = \frac{12}{5} + \frac{20}{5} = \boxed{(B) \frac{32}{5}}$.

Solution 4

[asy] unitsize(2cm); defaultpen(linewidth(.8pt)+fontsize(8pt));  draw((-1,-1)--(1,-1)--(1,1)--(-1,1)--cycle); draw((1,1)--(-1,0)); pair P=foot((1,-1),(1,1),(-1,0)); draw((1,-1)--P); draw(rightanglemark((-1,0),P,(1,-1),4));  label("$M$",(-1,0),W); label("$C$",(-0.1,-0.3)); label("$A$",(-0.4,0.7)); label("$B$",(0.7,0.4)); label("$P$",(-1,1),NW); label("$Q$",(1,1),NE); label("$R$",(1,-1),SE); label("$S$",(-1,-1),SW); label("$N$",P,NW); [/asy]

$MQ = \sqrt{2^{2} + 1^{2}} = \sqrt{5}$

since $\angle PQM + \angle PMQ = 90 = \angle PQM + \angle NQR$

therefore $\angle PMQ = \angle NQR$

and since $\angle MPQ = \angle QNR = 90$

therefore $\triangle MPQ \sim \triangle QNR$

therefore $\frac {[QNR]}{[MPQ]} = (\frac{QR}{QM})^{2}, [QNR] = [MPQ] \cdot (\frac{QR}{QM})^{2}$

$[MPQ] = \frac{1}{2} \cdot 2 \cdot 1 = 1$

$[QNR] = 1 \cdot (\frac{2}{\sqrt{5}})^{2} = 1 \cdot \frac{4}{5} = \frac{4}{5}$

Since the solution to the problem is $3[QNR] + 4$, the answer is $3(\frac{4}{5}) + 4 = \frac{12}{5} + \frac{20}{5} = \boxed{(B) \frac{32}{5}}$.

See Also

2009 AMC 10B (ProblemsAnswer KeyResources)
Preceded by
Problem 21
Followed by
Problem 23
1 2 3 4 5 6 7 8 9 10 11 12 13 14 15 16 17 18 19 20 21 22 23 24 25
All AMC 10 Problems and Solutions

The problems on this page are copyrighted by the Mathematical Association of America's American Mathematics Competitions. AMC logo.png